Hermitian matrix vector space over R proof

Click For Summary
Hermitian matrices can be proven to form a vector space over the real numbers (R) because the sum of two Hermitian matrices and the product of a Hermitian matrix by a real number both yield Hermitian matrices. The zero matrix is also included in this set, satisfying the requirements for a vector space. However, Hermitian matrices do not necessarily consist of real elements, as they can contain complex numbers. The discussion clarifies that while Hermitian matrices are not elements of R, they can still be considered a vector space over R. The key takeaway is that Hermitian matrices are indeed a vector space over R, not over C.
Jimena29
Messages
4
Reaction score
0

Homework Statement


I need to prove that the hermitian matrix is a vector space over R

Homework Equations





The Attempt at a Solution


I know the following:
If a hermitian matrix has aij = conjugate(aji) then its easy to prove that the sum of two hermitian matrices A,B give a hermitian matrix.
Multiplying a Hermitian matrix by a real number k will also give a hermitian matrix.
The zero vector is an element of the set of all Hermitian matrices.
So I can prove that hermitian matrices are a vector space, but what I`m stuck on is how a hermitian matrix is an element of real matrices??
How are complex numbers elements of R?
Thank you!
 
Physics news on Phys.org
A complex number is not an element of R, nor does a Hermitian matrix necessarily have real elements. You've already proved exactly what you need to prove. The Hermitian matrices are a vector space over R. They wouldn't be a vector space over C.
 
Oh, I thought I also had to prove that the matrix A is contained in R
Thank you very much!
 
Question: A clock's minute hand has length 4 and its hour hand has length 3. What is the distance between the tips at the moment when it is increasing most rapidly?(Putnam Exam Question) Answer: Making assumption that both the hands moves at constant angular velocities, the answer is ## \sqrt{7} .## But don't you think this assumption is somewhat doubtful and wrong?

Similar threads

  • · Replies 2 ·
Replies
2
Views
3K
  • · Replies 13 ·
Replies
13
Views
11K
  • · Replies 31 ·
2
Replies
31
Views
4K
  • · Replies 1 ·
Replies
1
Views
1K
  • · Replies 11 ·
Replies
11
Views
4K
  • · Replies 1 ·
Replies
1
Views
2K
  • · Replies 1 ·
Replies
1
Views
5K
  • · Replies 3 ·
Replies
3
Views
4K
  • · Replies 15 ·
Replies
15
Views
3K
  • · Replies 3 ·
Replies
3
Views
2K